LSAT and Law School Admissions Forum

Get expert LSAT preparation and law school admissions advice from PowerScore Test Preparation.

 Administrator
PowerScore Staff
  • PowerScore Staff
  • Posts: 8919
  • Joined: Feb 02, 2011
|
#27138
Complete Question Explanation
(See the complete passage discussion here: lsat/viewtopic.php?t=11403)

The correct answer choice is (A)

The passage concludes by stating that as long as the market and family roles stay as they currently are, women will continue to be disadvantaged in the labor market. To continue this idea, answer choice (A) addresses the other side of the gender consideration: men will continue to have more opportunities under the current conditions.
 sarae
  • Posts: 80
  • Joined: Aug 10, 2013
|
#10331
Why is answer choice D incorrect?
 Steve Stein
PowerScore Staff
  • PowerScore Staff
  • Posts: 1153
  • Joined: Apr 11, 2011
|
#10357
Hi sarae,

That's a Passage Expansion question that asks for an appropriate continuation of the discussion at the end of the passage.

Which direction was the author taking as the passage came to a close? Did you try to prephrase that one? It's always helpful to know how you went about responding to the question.

Let us know--thanks!

~Steve
 sarae
  • Posts: 80
  • Joined: Aug 10, 2013
|
#10566
My prephrase was something along the lines of how men wouldn't have been as seriously disadvantaged in the labor market as women were.

Both answers A and D seem to address the fact that men will have more opportunities.

I guess D is wrong because it addresses "men with primary child rearing responsibilities," while the end of the passage talking about family roles on the basis of gender? (Which means women have the primary responsibilities?)
 Steve Stein
PowerScore Staff
  • PowerScore Staff
  • Posts: 1153
  • Joined: Apr 11, 2011
|
#10573
Hey sarae,

You got it--it would be a strange turn for the author to go back to the subject of men with primary child-rearing responsibilities at that point (from the beginning of the second paragraph), but answer choice A continues the flow of the discussion that takes place at the end of the passage.

I hope that's helpful!

~Steve
 T.B.Justin
  • Posts: 194
  • Joined: Jun 01, 2018
|
#60612
I thought that 'B' was correct; the passage says that culturally women bear the pressure more than men, and gives examples of employment barriers faced by women with primary child-care responsibilities. It goes on to to explain that even in families with both working parents that the role of primary child-care responsibility falls predominantly on the woman. The author then gives his conclusion on how in this type of labor market these problems that more often disadvantage women than men will remain. I thought 'B' would follow from that.
 Brook Miscoski
PowerScore Staff
  • PowerScore Staff
  • Posts: 418
  • Joined: Sep 13, 2018
|
#62687
Justin,

(A), the credited response, does seem a bit idiotic--after all, how does it do more than repeat the last sentence of the passage? I guess it's hard to argue that the passage can't follow up by simply repeating itself through a corollary.

The problem with (B) is that the passage contained no discussion of the willingness or motives of men, so (B) cannot be selected.
 T.B.Justin
  • Posts: 194
  • Joined: Jun 01, 2018
|
#62697
Hey Brook,

Thanks, you helped me realize I brought my own beliefs about rejecting traditional stereotypes of family roles into my thought process for this passage.
 anthonychernandez
  • Posts: 13
  • Joined: May 06, 2019
|
#65023
Can you explain why E is incorrect?

I thought "institutions" in E was broad enough to include the job market and family structure, both of which are identified as widening the gap for career opportunities.

I didn't choose the correct answer A because 1. "most" scared me away and 2. although there's more discussion of career opportunity disadvantages than salary disadvantages for women, saying that this meant that men "remain better able to enjoy ... opportunities" seemed like inferring too much.
 Rachael Wilkenfeld
PowerScore Staff
  • PowerScore Staff
  • Posts: 1358
  • Joined: Dec 15, 2011
|
#65062
Hi Anthony,

I think your reading of the word "institutions" is fine here. The problem is that the passage does not support the idea that the gap is widening. It supports that there is a gap, that the gap is not improving, but there is not anything that we can point to show a widening gap.

For answer choice (A), we can support it with the passage. The passage's main point is that mothers are disadvantaged in the work force because of societal institutions that are not compatible with family expectations. It also supports that women are expected to bear the brunt of those expectations. Therefore, we can draw that most man would be better able to enjoy the labor market.

Hope that helps.
Rachael

Get the most out of your LSAT Prep Plus subscription.

Analyze and track your performance with our Testing and Analytics Package.